You are on page 1of 77

Q.

1
John is a 56 year old regular patient of your rural clinic and is booked for an elective total knee replacement in two weeks time at the closest regional
hospital. He was asked by the hospital anaesthetist to attend your clinic today for a routine pre-operative ECG and he plans to have blood tests
performed later today. The nurse hands you John’s ECG for review prior to faxing it to the regional hospital. You are concerned about the ECG and
check John’s file to review his past medical history and medications.

Which medications from the list below are mostly likely to be responsible for these ECG changes? Choose one (1) option.

Digoxin

ACE inhibitor and spironalactone

Atenolol
Quetiapine and amitriptyline

Carbamazepine

Your answer is correct.

General Feedback:
The correct response is: ACE inhibitor and spironolactone.
This combination of medications can predispose patients to hyperkalaemia. This ECG shows tall peaked T
waves which can be a sign of hyperkalaemia. Other ECG signs of hyperkalaemia include:
Prolonged PR segments
Loss of P waves
Bradycardia
Sine wave
Reference and image source:
Buttner R, Burns E. Life in the Fast Lane. April 2021. https://litfl.com/hyperkalaemia-ecg-library/ (Accessed
June 2021)
ETG complete by therapeutic guidelines. Electrolyte abnormalities. Hyperkalaemia. Updated March 2021.
Available from: https://tgldcdp.tg.org.au/viewTopic?topicfile=electrolyte-abnormalities&guidelineName=Othe
r#toc_d1e1118

Q.2
Luke Rein, a 48 year old man, presents to your GP clinic with a 3 day history of intermittent sharp chest pain. He reports the chest pain was worse
when lying in bed last night, with improvement when sleeping upright on his lounge chair. He recalls having a sore throat two weeks ago.

Luke’s body mass index (BMI) is 27 kg/m2 , blood pressure is 122/80mmHg, jugular venous pressure is not elevated and heart rate is 65 beats per
minute, regular, heart sounds are dual, with no murmurs. He does not have any tenderness on palpation of his chest.
After confirming your suspected diagnosis, what three (3) treatment options would you advise Luke to undertake? Choose one (1) option.
Colchicine; aspirin; restriction of strenuous exercise until symptoms have resolved

Aspirin; colchicine; restriction of all exercise for 12 months

Colchicine; prednisolone; exercise as desired

Prednisolone, aspirin; restriction of strenuous exercise for 6 months

Paracetamol; benzyl penicillin; no activity restriction is required

Your answer is incorrect.

General Feedback:
The correct response is: Colchicine; aspirin; restriction of strenuous exercise until symptoms have resolved.
This is a straightforward presentation of pericarditis and requires candidates to know the three main
principles of management. Aspirin can be used initially at higher doses (750mg to 1000mg TDS) for the first
1 – 2 weeks. The dose is then gradually tapered down. It is important to continue colchicine for 3 months to
prevent recurrences and to avoid strenuous exercise until symptom resolution. However, in competitive
athletes, strenuous exercise needs to be avoided for 3 months after the resolution of symptoms.
References:
Rahman A, Saraswat A. Pericarditis. Australian Family Physician. 2017 Nov;46(11):810 Available from: https://
www.racgp.org.au/download/Documents/AFP/2017/November/AFP-2017-11-Focus-Pericarditis.pdf (Accessed
Sept 2022).
eTG complete by Therapeutic Guidelines. Cardiology: Pericarditis – Management of pericarditis; March 2018.
Available from: https://tgldcdp.tg.org.au/viewTopic?topicfile=pericarditis&guidelineName=Cardiovascular#to
c_d1e101 (Accessed Sept 2022).
Correct answers is: Colchicine; aspirin; restriction of strenuous exercise until symptoms have resolved

Q3
Q.3
Chris Mills is a 58 year old man who has recently been discharged from hospital following an inferior myocardial infarction that was managed with a
drug eluding stent. He presents for a review of his medication. He states although he saw a pharmacist prior to discharge he is confused about his
long-term medications and the plan for his ongoing care.

Which of the following statements is correct? Choose one (1) option.

He should be on the lowest possible dose of a statin indefinitely

Dual antiplatelet therapy with aspirin plus a P2Y12 inhibitor should be continued for 6-12 months

ACE inhibitors are recommended but have not been shown to improve survival

Only high risk patients should be referred for cardiac rehabilitation following a myocardial infarction

A potassium sparing diuretic is recommended

Your answer is correct.

General Feedback:
The correct response is: Dual antiplatelet therapy with aspirin plus a P2Y12 inhibitor should be continued for
6-12 months
Patients who have had a coronary artery stent inserted are at risk of acute stent thrombosis, which is a rare
but serious complication of coronary artery stents. Drug-eluting stents are associated with a lower rate of
stent thrombosis than bare-metal stents, and the second generation of drug-eluting stents are associated
with a lower rate of stent thrombosis than the first generation.
To minimise the risk of stent thrombosis, patients who have a coronary artery stent are routinely treated with
a period of dual antiplatelet therapy (DAPT) (low-dose aspirin and a P2Y12 inhibitor [clopidogrel, prasugrel or
ticagrelor]). The duration of DAPT is primarily determined by the type of stent used, but also takes into
consideration individual patient-related factors that affect the risk of bleeding or thrombosis. After insertion
of a bare-metal coronary artery stent, DAPT is recommended for 6 weeks. Drug-eluting stents take longer to
endothelialise than bare-metal stents, so DAPT is usually recommended for 6 to 12 months after insertion of
a drug-eluting coronary artery stent.
Cardiac rehabilitation is recommended for all patients hospitalised with Acute Coronary Syndrome. Survival
following a myocardial infarction is improved with the use of ACE inhibitors. Unless there is a history of
intolerance or a contraindication, patients should take the highest tolerated dose of a statin indefinitely
following hospitalisation for Acute Coronary Syndrome.

References:
Chew DP et al. National Heart Foundation of Australia & Cardiac Society of Australia and New Zealand:
Australian clinical guidelines for the management of acute coronary syndromes 2016. Heart, Lung and
Circulation, 25 (2016):895-951
https://www.heartfoundation.org.au/images/uploads/publications/
Clinical_Guidelines_for_the_Management_of_Acute_Coronary_Syndromes_2016.pdf
Therapeutic Guidelines eTG Complete. Cardiovascular: Antithrombotic therapy after endovascular and
cardiac interventions. 2018.
https://tgldcdp.tg.org.au/viewTopic?etgAccess=true&guidelinePage=Cardiovascular&topicfile=antithromboti
cs-after-cardiac-interventions

Q.4
Jonathon Brim is a 49 year old male who has been commenced on haloperidol by his treating psychiatrist as treatment for his bipolar disorder. He
presents with a racing heart and you perform an electrocardiogram in your clinic.

What does his electrocardiogram reveal? Choose one (1) option.


Tachycardia with QT prolongation

Atrial fibrillation with rapid ventricular rate

Acute ST elevation myocardial infarction

Left bundle branch block

Tachycardia only
Your answer is correct.

General Feedback:
The correct response is: Tachycardia with QT prolongation
This is a potential adverse effect of haloperidol. Antipsychotics can increase the QT interval and the risk of
arrhythmia and sudden death. It is recommended that patients on long-term antipsychotics have an
echocardiogram performed annually (and when clinically indicated).

References:
Australian Medicines Handbook. Haloperidol. https://amhonline.amh.net.au/chapters/psychotropic-drugs/anti
psychotics/haloperidol (Accessed November 2021) (Requires paid subscription).
Life in the Fast Lane. Drugs Causing QT Prolongation. https://litfl.com/drugs-causing-qt-prolongatio
n/ (Accessed November 2021).

Q.5
Frank Moy is a 76 year old male who presents with fatigue, intermittent palpitations and shortness of breath at rest. He is currently on Perindopril 5mg
daily for hypertension and takes no other regular medications. He has a past history of peripheral arterial disease and a transient ischaemic attack
(TIA) 2 years ago.

On examination you note that he has an irregularly irregular heart rate.

You suspect that Frank has atrial fibrillation and you arrange an electrocardiogram which confirms this diagnosis. You base your decision for
anticoagulation for stroke prevention on his CHA2DS2-VA score.

What is Frank's score? Choose one (1) option.

6
5

Your answer is correct.

General Feedback:
The correct response is: 6
The 2018 Australian Clinical Guidelines for the Diagnosis and Management of Atrial Fibrillation recommends
the use of the CHA2DS2–VA score to assess stroke risk in patients with atrial fibrillation. This patient’s score
is 6:
History of hypertension: 1 point
Age ≥75 years: 2 points
History of prior TIA: 2 points
Vascular disease (peripheral arterial disease): 1 point

Reference
National Heart Foundation of Australia and the Cardiac Society of Australia and New Zealand. Australian
Guidelines for the Diagnosis and Management of Atrial Fibrillation 2018. https://www.heartlungcirc.org/articl
e/S1443-9506(18)31778-5/fulltext

Q.6
You are a GP in a rural town and are looking after Samantha Wallis, a 53 year old woman, who has been recently diagnosed with heart failure. She
travelled to see a visiting cardiologist in a nearby town who performed an echocardiogram which showed a left ventricular ejection fraction of 35%.
The cardiologist has started her on enalapril 2.5mg orally daily and bisoprolol 1.25mg orally daily and asked you to titrate these up depending on
tolerance. Her recent blood tests show normal renal function.

What other medication would you expect the cardiologist has commenced for Samantha at this stage to improve her survival and reduce her risk of
hospitalisation from her heart failure? Choose one (1) option.
Spironolactone 25mg orally daily

Diltiazem 180mg orally daily

Frusemide 20mg orally daily

Sacubitril + valsartan 49mg+51mg orally twice daily

Verapamil 160mg orally daily

Your answer is correct.

General Feedback:
The correct answer is: Spironolactone 25mg orally daily
Drugs that improve outcomes in heart failure with reduced ejection fraction (HFrEF) include angiotensin
converting enzyme inhibitors (ACEI), angiotensin II receptor blockers (ARB), beta blockers, aldosterone
antagonists and angiotensin-receptor neprilysin inhibitors (ARNI) (eg sacubitril+valsartan). Good quality
clinical evidence shows that these drugs improve survival and reduce heart failure hospitalisation.
An ARNI should only be considered in patients with symptomatic heart failure despite optimal therapy with
an ACEI or ARB and beta blocker therapy. It is not indicated for Samantha at this stage.
Loop diuretics can be used in patients with heart failure with reduced ejection fraction (HFrEF) to reduce the
signs and symptoms of congestion (e.g. breathlessness, peripheral oedema) and improve exercise
tolerance. But there is no high-level evidence that loop diuretics reduce mortality or morbidity.
Negative inotropic effect of non-dihydropyridine calcium channel blockers –verapamil and diltiazem may
depress cardiac function. These should be used with caution and do not have evidence for improving
morbidity or mortality in patients with heart failure.
Reference:
Therapeutic Guidelines eTG complete. Cardiovascular: Heart failure. 2018. Available from: https://tgldcdp.tg.
org.au/viewTopic?topicfile=heart-failure&guidelineName=Cardiovascular&topicNavigation=navigateTopic

Q7
Q.7
Simon Popper, a 34 year old male patient, presents with a history of chest pain, shortness of breath and fever. He states the sharp pain is located over
the left side of his chest and is made worse by inspiration and lying flat. You perform an electrocardiogram which supports your suspected diagnosis
of pericarditis.

What could you expect to see on Simon's electrocardiogram? Choose one (1) option.

PR depression and diffuse ST elevation which can be followed by T wave flattening and inversion

Loss of the P wave with tall tented T waves

PR elevation and diffuse ST elevation which can be followed by tall tented T waves

PR depression and diffuse ST depression which can be followed by QRS widening

PR depression and diffuse ST elevation which can be followed by peaked T waves

Your answer is incorrect.

General Feedback:
The correct response is: PR depression and diffuse ST elevation which can be followed by T wave flattening
and inversion

Pericarditis is classically associated with ECG changes that evolve through four stages.
Stage 1 – widespread ST elevation and PR depression with reciprocal changes in aVR (occurs during the
first two weeks)
Stage 2 – normalisation of ST changes; generalised T wave flattening (1 to 3 weeks)
Stage 3 – flattened T waves become inverted (3 to several weeks)
Stage 4 – ECG returns to normal (several weeks onwards)
However, note that less than 50% of patients progress through all four of these classical stages and
evolution of changes may not follow this typical pattern.

Reference:
Life in the Fast Lane. Pericarditis ECG Changes. Updated March 2021. Available from:
https://litfl.com/pericarditis-ecg-library/ (Accessed September 2022).

Correct answers is:


PR depression and diffuse ST elevation which can be followed by T wave flattening and inversion

Q.8 Fred, a seven year old boy is brought to you by his parents who say that he has been complaining of trouble with his vision at school. On
examination his blood pressure is 145/90mmHg. You take further history and perform further examination to determine that Fred is likely to be
suffering with either primary or essential hypertension.

Which feature in this list is most likely to fit with essential hypertension? Choose one (1) option.

Abdominal bruit

Ambiguous genitalia

Delayed femoral pulses

Diastolic hypertension

History of umbilical artery catheterisation

History of urinary tract infections


Malignant hypertension

Nocturnal hypertension

Family history of hypertension

Oedema

Prepubertal child

Resistant hypertension despite the concurrent use of 3 anti-hypertensives from different classes at adequate doses

Tachycardia, sweating, headache and flushing

Your answer is correct.

General Feedback:
The correct response is: Family history of hypertension

Obesity and a family history of hypertension make essential rather than secondary hypertension more likely.
The phenotype of essential hypertension can present as early as 3 years of age and is the predominant form
of hypertension in children after age of 6 years.
References:
1. The Royal Childrens Hospital Melbourne. Hypertension in children and adolescents. Updated 2021.
Available from: https://www.rch.org.au/clinicalguide/guideline_index/hypertension/ (Accessed December
2022)
Gupta-Malhotra M et al. Essential Hypertension vs. Secondary Hypertension Among Children. American
Journal of Hypertension 2015; 28(1): 73–80.
https://www.ncbi.nlm.nih.gov/pmc/articles/PMC4318949/ (Accessed December 2022)

Q9
Q.9
Brian Lucas is a 68 year old retired bricklayer. He presents to your clinic with pain in his right leg for the past three months. Brian describes this pain
as a deep ache in his calf which starts after he has walked for about ten minutes when he goes for his usual morning walks. He often has had to stop
and rest on a park bench for several minutes before he is able to resume walking. Brian has not had any pain at night-time or when he is resting. His
past medical history includes osteoarthritis in his back and knees, hypertension and gout. He is an ex-smoker and drinks 3 standard drinks of beer a
week.
An examination, his gait is normal, there is stiffness and crepitus in both his knees and limited lumbar forward flexion. His dorsalis pedis and
posterior tibial pulses are faintly palpable in the right foot. He has a negative straight leg raise, normal power and sensation in his lower limbs. His
right calf is not tender or swollen.
Which of the following is the next most appropriate investigation to perform? Choose on (1) option.

Duplex arterial ultrasound of the lower limbs

Computed tomography (CT) lumbosacral spine

Ankle-brachial index

Magnetic resonance imaging (MRI) scan of his lumbosacral spine

Duplex venous ultrasound of the lower limbs

Your answer is correct.

General Feedback:
The correct response is: Ankle-brachial index
Brian’s symptoms are consistent with intermittent claudication. He has several risk factors for this including
his age, smoking history and hypertension. An ankle brachial index (ABI) would reveal a reading of <0.9 in
intermittent claudication. A duplex arterial ultrasound of the lower limbs can be considered if surgical
intervention for peripheral vascular disease is being considered in the setting of an abnormal ABI.
It is common for patients with intermittent claudication to also have comorbid musculoskeletal disease and
differentials for calf pain can include neurogenic claudication. However, in Brian’s case, apart from stiffness
in the affected joints, there is no suggestion that he has neurogenic claudication as his straight leg raise is
negative and there are no neurological findings in his lower limbs.
Deep vein thrombosis is also a differential diagnosis for unilateral calf pain. However, in the clinical stem
provided, his pain is on exertion of a fixed distance and not at rest. In addition, he does not have any clinical
examination findings that support a diagnosis of venous thromboembolism.
Reference:
Aitken SJ. Peripheral artery disease in the lower limbs: The importance of secondary risk prevention for
improved long-term prognosis. Aust J Gen Pract [Internet]. 2020 May. Available from: https://www1.racgp.or
g.au/ajgp/2020/may/peripheral-artery-disease-in-lower-limbs (Accessed July 2021).

Q.10
Raylene Gray is a 64 year old female who is a new patient to your clinic. She has just moved into the area. She brings along her current medications
to show you and you begin to enter them into your medical software. Raylene presents a box of verapamil, but when you ask about the reason why
she is taking this medication she says she is not sure. Based on her other medications you decide to screen her for contraindications to this drug.
Non-dihydropyridine calcium channel blockers are contraindicated in which group of patients? Choose one (1) option.

Hypertensive patients with heart failure with reduced ejection fraction

Hypertensive patients with diabetes mellitus

Hypertensive patients with ischaemic heart disease

Hypertensive patients with peripheral vascular disease

Hypertensive patients with renal disease

Your answer is correct.

General Feedback:
The correct response is: Hypertensive patients with heart failure with reduced ejection fraction
Non-dihydropyridine calcium channel blockers (e.g. diltiazem, verapamil) can cause depression of cardiac
function in patients with heart failure with reduced ejection fraction and are thus, contraindicated (unless
under specialist supervision) as per the Australian Medicines Handbook. This can also occur with
dihydropyridines which are recommended to be used with caution.
Reference:
Australian Medicines Handbook. Calcium channel blockers. https://amhonline.amh.net.au/chapters/cardiova
scular-drugs/antihypertensives/calcium-channel-blockers (Accessed December 2021).

Q.11
Francis Boland is a 55 year old man with a history of hypertension and hypercholesterolaemia who had an anterior myocardial infarction last year. His
regular medications include ramipril 5mg orally daily, amlopidine 5mg orally daily, rosuvastatin 10mg orally daily, aspirin 100mg orally daily and
omeprazole 20mg orally daily.

Francis presents today stating that both of his legs are swollen every day. He denies any shortness of breath on
exertion or breathlessness at night. His current blood pressure is 130/85mmHg. There is mild bilateral lower leg
swelling (left = right) and there is no calf tenderness on compression.
As well as monitoring his blood pressure and leg swelling, which one of the following is the most appropriate next step in Francis' management?
Choose one (1) option.

Cease his amlodipine

Add frusemide at a dose of 20mg orally daily

Increase his ramipril from 5mg to 10mg orally daily

Add a thiazide diuretic in the form of indapamide 1.5mg orally daily

Cease his rosuvastatin

Your answer is incorrect.


General Feedback:
The correct response is: Cease his amlodipine
The most likely cause for his swollen legs is the dihydropyridine calcium channel blocker (amlodipine) as
these drugs commonly cause peripheral oedema due to redistribution of extracellular fluid (rather than fluid
retention). Commencing the patient on a diuretic in this situation will not resolve the swelling and may place
the patient at risk of volume depletion. You would cease this medication in the first instance and continue to
monitor his blood pressure and see if the swelling resolved.
Reference:
Australian Medicines Handbook. Amlodipine. Available from: https://amhonline.amh.net.au/chapters/cardiova
scular-drugs/antihypertensives/calcium-channel-blockers/amlodipine
Correct answers is: Cease his amlodipine

Q.12
Ralph Jenkins is a 58 year old man who presents with an episode of severe chest pain at rest, in the context of a 2 week history of exertional chest
pain. He has a history of hypertension and type 2 diabetes mellitus. He also has a family history of ischaemic heart disease with his father having
coronary artery bypass grafting (CABG) at 54 years of age. Ralph is an ex-smoker with a 40 pack year smoking history. The chest pain was located
over the left side of his chest and he described a dull and heavy sensation, associated with shortness of breath and diaphoresis. His
electrocardiogram demonstrated sinus rhythm with ST elevation in leads II, III and aVF

Which of the following coronary arteries is most likely occluded in Ralph's case? Choose one (1) option.

Left anterior descending

Left circumflex artery

Right coronary artery

Diagonal arteries

Right marginal artery


Your answer is correct.

General Feedback:
The correct response is: Right coronary artery
Leads II, III and aVF are the 'inferior' leads. Approximately 40 - 50% of myocardial infarctions are inferior
myocardial infarctions. About 80% of inferior myocardial infarctions are caused by occlusion of the right
coronary artery. Less commonly, the dominant left circumflex or a "type III" or "wraparound" left anterior
descending artery may be involved. The latter involves an unusual pattern of concomitant anterior and
inferior ST elevation.
Reference:
Life in the fast lane. ECG library. Inferior STEMI. Available from: https://litfl.com/inferior-stemi-ecg-library/

Q.13
61 year old school teacher, Agnes Hopkins attends your clinic for annual review of her non-valvular atrial fibrillation (AF). She denies any palpitations
or dizziness. Agnes has a history of obstructive sleep apnoea, obesity and hypothyroidism. She is on thyroxine 125micrograms daily and her last
thyroid function test a year ago showed that this dose was appropriate at the time. Agnes takes no other regular medications.
You perform an electrocardiogram which shows the following:
What is the most appropriate AF management option for Agnes until your next scheduled review in 12 months’ time? Choose one (1) option.

Commence oral amiodarone 200mg orally daily

Focus on risk factor control (e.g. managing obstructive sleep apnoea, maintaining a euthyroid state, weight loss)

Commence aspirin 100mg orally daily and manage her risk factors (e.g. obstructive sleep apnoea, hypothyroidism,
obesity)
Commence dual therapy with aspirin 100mg orally daily and rivaroxaban 5 mg orally twice daily

Refer Agnes for a catheter ablation

Your answer is incorrect.

General Feedback:
The correct response is: Focus on risk factor control (e.g. obstructive sleep apnoea, hypothyroidism,
obesity)
Agnes is at low risk of thromboembolic complications at age 61 years with stable, asymptomatic AF and her
CHA2DS2-VASc score is 1 (she gets 1 point for being female). According to the CHA2DS2-VASc female sex
only increases stroke risk when other risk factors are present. It is reasonable to focus on risk factor
reduction at this stage as obesity and sleep apnoea can both be contributors of atrial fibrillation.
Amiodarone infusions can be used for rhythm control and oral amiodarone can be used for rate control in
patients with left ventricular dysfunction. However, Agnes is asymptomatic from her AF and amiodarone may
exacerbate her hypothyroidism. Catheter ablation is not indicated in this setting. There are also significant
anaesthetic risks associated with obesity and obstructive sleep apnoea. Aspirin does not have adequate
anticoagulant effect in AF. The use of direct-acting anticoagulants such as rivaroxaban is not indicated for a
CHA2DS2-VASc score of 0 in men and 1 in women. Dual therapy with aspirin and an anticoagulant would
increase bleeding risks without providing additional anticoagulant protection.
Reference:
McCallum CJ et al. Atrial fibrillation: an update on management. Australian Prescriber 2019; 42: 186-191.
Available from: https://www.nps.org.au/australian-prescriber/articles/atrial-fibrillation-an-update-on-manage
ment
Therapeutic Guidelines. Atrial Fibrillation [Internet]. 2018 Mar [update 2019 Dec; cited 2021 Nov]. Available
from: https://tgldcdp.tg.org.au/viewTopic?topicfile=atrial-fibrillation&guidelineName=Cardiovascular&topicN
avigation=navigateTopic#toc_d1e149 (paid subscription required)
Burns E, Buttner R, Atrial Fibrillation. Life in the Fast Lane [Internet]. 2021 Jun 6 [cited 2021 Nov 1]. Available
from: https://litfl.com/atrial-fibrillation-ecg-library/
Correct answers is:
Focus on risk factor control (e.g. managing obstructive sleep apnoea, maintaining a euthyroid state, weight
loss)

Q 14
Q.14
Brian Krank is a 67 year old male who presents to see you for review after recently being diagnosed with heart failure with reduced ejection fraction
after presenting with a 6 month history of cough and shortness of breath. On examination, you note that he has bilateral fine inspiratory crepitations
and mild shortness of breath on exertion. He has significant peripheral oedema to the level of the knees bilaterally.

You discuss his medications with him and then discuss non-pharmacological management.
What advice do you provide to Brian regarding fluid restriction and dietary sodium intake? Choose one (1) option.

Brian should consume < 2 grams sodium / day and restrict his fluids to < 1.5L / day

Current guidelines do not support specific sodium or fluid restriction in heart failure management

Brian should consume < 5 grams sodium / day and restrict his fluids to < 2L / day

Brian should consume < 5 grams sodium / day and manage his fluids according to his daily weighs

Brian should consume < 1 gram sodium / day and restrict his fluids to < 1.5L / day

Your answer is incorrect.

General Feedback:
The correct response is: Brian should consume < 2 grams sodium / day and restrict his fluids to < 1.5L / day.
This question is testing your knowledge of the 2018 Heart Failure guidelines. There is no evidence beyond
expert opinion to support the amount of sodium intake for patients with heart failure, so the current advice is
to apply the National Heart Foundation Australia general population recommendation regarding sodium
intake which is < 2g/day.
Regarding fluid restriction, the guidelines recommend:
For patients with overt congestion: restrict to 1.5L fluid / day
If the patient’s weight increases by 2kg over 2 days: recommend GP review (consider a temporary
increase in diuretics depending on haemodynamic status, renal function and electrolytes)
Consider a sliding-scale of diuretics for patients who are competent in self-management

Reference:
National Heart Foundation & Cardiac Society of Australia and New Zealand. Guidelines for the Prevention,
Detection & Management of Heart Failure in Australia 2018. Heart, Lung & Circulation 2018; 27(10): 1123 –
1208. Available from: https://www.heartlungcirc.org/article/S1443-9506(18)31777-3/fulltext
Correct answers is: Brian should consume < 2 grams sodium / day and restrict his fluids to < 1.5L / day

Q.15
Mary George, a 67 year old female, attends your clinic to discuss her husband’s recent ischaemic stroke secondary to previously undiagnosed atrial
fibrillation. Mary asks you if she should be checked for atrial fibrillation as she is concerned about her own risk of stroke. She has no symptoms of
concern including no palpitations, shortness of breath or fatigue. Mary has no significant past medical history and she takes no regular medications.

Based on current guidelines, what advice about screening do you provide to Mary? Choose one (1) answer.

Advise Mary that screening for silent atrial fibrillation is not supported by current guidelines

Advise Mary that screening for silent atrial fibrillation is recommended in patients aged 65 years and older

Advise Mary that screening for silent atrial fibrillation is recommended in patients aged 75 years and older

Advise Mary that screening for silent atrial fibrillation is recommended in patients aged over 60 years

Advise Mary that screening for silent atrial fibrillation is recommended in patients aged over 70 years

Your answer is incorrect.

General Feedback:
The correct response is: Advise Mary that screening for silent atrial fibrillation is recommended in patients
aged 65 years and older.
The 2018 Australian atrial fibrillation guidelines recommend opportunistic screening for silent atrial
fibrillation in patients aged ≥65 years. There is strong evidence to suggest that 10% of ischaemic strokes are
related to previously undiagnosed atrial fibrillation.

References:
National Heart Foundation of Australia and Cardiac Society of Australia and New Zealand. Australian Clinical
Guidelines for the Diagnosis and Management of Atrial Fibrillation 2018. Available from: https://www.heartlun
gcirc.org/article/S1443-9506(18)31778-5/fulltext#sec0090
Correct answers is:
Advise Mary that screening for silent atrial fibrillation is recommended in patients aged 65 years and older

Q.16
Bill Weatherill is a 50 year old man who was recently admitted to hospital with an acute myocardial infarction. He was treated successfully with a drug
eluting stent and had an uneventful recovery with no arrhythmias. His echocardiogram showed left ventricular ejection fraction of 38%. He comes to
see you on discharge from hospital.

What combination of medications is Bill most likely to have been discharged on to prevent a further myocardial infarction? Choose one (1) response.

Aspirin, ticagrelor, atorvastatin, carvedilol, fosinopril

Aspirin, clopidogrel, atorvastatin, propranolol, fosinopril

Aspirin, ticagrelor, atorvastatin, carvedilol, spironolactone

Aspirin, clopidogrel, atorvastatin, carvedilol, sacubitril+valsartan

Aspirin, ticagrelor, atorvastatin, carvedilol, ditiazem

Your answer is correct.

General Feedback:
The correct response is: Aspirin, ticagrelor, atorvastatin, carvedilol, fosinopril
In general principals, the drug therapy for the secondary prevention of atherosclerotic cardiovascular
disease usually consists antiplatelet drug(s) with a of a combination of a statin, an angiotensin converting
enzyme inhibitor (ACEI), and unless contraindicated, a beta-blocker.
Following an acute coronary syndrome, dual antiplatelet therapy with aspirin and a P2Y12 inhibitor
(clopidogrel, prasugrel or ticagrelor) is usually recommended for 12 months.
Statin therapy reduces cardiovascular morbidity and mortality in patients with established atherosclerotic
CVD, and high dose therapy should be given irrespective of the patient’s lipid levels.
Angiotensin converting enzyme inhibitors (ACEI) prevent cardiovascular events including myocardial
infarction, stroke and death in patients with atherosclerotic CVD; these effects may be independent of blood
pressure reduction.
While beta-blocker therapy should be started immediately following an acute coronary syndrome (unless
contraindicated). Patients with reduced left ventricular systolic function (left ventricular ejection fraction 40%
or less) or evidence of ongoing ischaemia (including stable angina) are more likely to benefit from long-term
beta-blocker therapy. Use one of the beta blockers shown to be effective in heart failure with reduced
ejection fraction (HFrEF) (bisoprolol, carvedilol, metoprolol succinate or nebivolol).
ARNIs (valsartan with sacubitril) play an important role in the management of heart failure, but is not the
first-line treatment. First-line is with an ACE-I or ARB. The heart failure guidelines state that: "In patients with
persistent symptomatic HFrEF despite optimal ACEI or ARB and beta-blocker therapy, replacement of the
ACEI or ARB with sacubitril+valsartan may be considered."

Reference:
National Heart Foundation of Australia and the Cardiac Society of Australia and New Zealand. Australian
Clinical Guidelines for the Management of Acute Coronary Syndromes 2016. https://www.heartfoundation.or
g.au/getmedia/6132a46d-5cfc-4cec-a9da-2ff380179bb1/Clinical_Guidelines_for_the_Management_of_Acute_
Coronary_Syndromes_2016.pdf(Accessed November 2022)
Therapeutic Guidelines eTG Complete. Cardiovascular: Secondary prevention of atherosclerotic
cardiovascular events. March 2018. Available from: https://tgldcdp.tg.org.au/viewTopic?topicfile=secondary-
prevention-cardiovascular-disease&guidelineName=Cardiovascular&topicNavigation=navigateTopic#toc_d1
e76 (Accessed November 2022)

Q.17
Phil is a 62 year old Caucasian man who has returned for a review of his blood pressure. You saw him three months ago for a general check up and
his blood pressure was 158/95 at the time. After arranging some bloods, you calculated him to have a moderate cardiovascular risk. He has no family
history of premature cardiovascular disease. You counselled him on lifestyle measures to improve his blood pressure and arranged to review him in 3
months time.

At your review today, his cardiovascular risk remains unchanged. However you still feel concerned about his blood pressure.
According to the National Heart Foundation of Australia 'Guideline for the diagnosis and management of hypertension in adults', which one of the
following measurements would indicate that an antihypertensive should be commenced for Phil?
Please choose one (1) from the following:

≥ 140/90 mmHg

≥ 150/90 mmHg

≥ 150/100 mmHg

≥ 160/90 mmHg

≥ 160/100 mmHg

Your answer is correct.

General Feedback:
According to the National Heart Foundation of Australia Guideline for the diagnosis and management of
hypertension in adults: for patients with moderate absolute cardiovascular risk with persistent blood
pressure upon repeat review of >140mmHg systolic and/or >90mmHg diastolic, antihypertensive therapy
should be started.
National Heart Foundation of Australia. Guideline for the diagnosis and management of hypertension in
adults. 2016. Available at: https://www.heartfoundation.org.au/Conditions/Hypertension (Accessed November
2022)

Q.18
Luci Read is a 68 year female who attends your clinic for a drivers licence medical examination. She has a past history of hypertension and aortic
stenosis. Her last echocardiogram was conducted six months ago. Her most recent blood tests from two weeks ago reveal a normal full blood count,
renal function, liver function and thyroid stimulating hormone level.
While you are examining Luci today you note that she has an irregularly irregular pulse. You ask your practice nurse to perform an electrocardiogram
which reveals atrial fibrillation with a rate of 90 beats per minute. You then calculate Luci’s CHA2DS2-VASc score which is 3 and her HAS-BLED score
which is 1.

You commence Luci on oral atenolol for rate control. What is the next most appropriate step in the management of her atrial fibrillation? Choose one
(1) option.

Refer Luci to a cardiologist for further advice before commencing any other medication

Commence Luci on 100mg oral aspirin daily

Commence Luci on oral warfarin titrating to an International Normalised Ratio (INR) of 2.5 – 3.5

Commence Luci on rivaroxaban 20mg orally daily

Commence Luci on clopidogrel 75mg orally daily

Your answer is correct.

General Feedback:
The correct response is: Commence Luci on rivaroxaban 20mg orally daily
This question is testing your knowledge of the prevention of stroke and thromboembolic events in the
setting of atrial fibrillation. Anticoagulation with either warfarin or a non-vitamin K antagonist oral
anticoagulant (NOAC) is recommended for this indication. Anti-platelet therapy with aspirin or
P2Y12 inhibitors (dual antiplatelets) are not recommended for the prevention of stroke or thromboembolic
events in patients with atrial fibrillation.
Warfarin is recommended to be used in patients with mechanical heart valves or moderate or severe mitral
stenosis. NOACs should not be used in these patients or in those with severe kidney disease.
As stated in eTG, “there is no clear evidence that other valvular heart disease (eg mitral regurgitation, aortic
valve disease) needs to be considered when choosing an anticoagulant or in estimating stroke risk. Patients
with these conditions should be treated as for patients with atrial fibrillation who do not have moderate or
severe mitral stenosis, or a mechanical heart valve.”
Given that Luci has a CHA2DS2-VASc score of 3 and a low HAS-BLED score, it would be appropriate to
recommend a NOAC in her case, such as oral rivaroxaban. Warfarin would be an acceptable alternative,
although the target INR would be 2 – 3 (rather than 2.5 – 3.5) and she would require regular INR monitoring.
Reference:
Therapeutic Guidelines (eTG). Atrial fibrillation. Amended December 2019. Available from: https://tgldcdp.tg.o
rg.au/viewTopic?topicfile=atrial-fibrillation (Accessed March 2022; Paid subscription required).

Q.19
George is a 71 year old patient who is brought in by his wife with a two day history of feeling fatigued, short of breath and dizzy. He has also noticed
occasional palpitations and mild central chest discomfort. George denies any recent illnesses, fever, cough or sputum production. You note that he
has an irregular pulse so you ask your practice nurse to perform a set of observations in addition to an urgent electrocardiogram which is below. What
is the most likely diagnosis? Choose one (1) option.
Atrial flutter

Lateral non-ST elevation myocardial infarction

Ventricular fibrillation

Atrial fibrillation with rapid ventricular response

Ventricular tachycardia

Your answer is correct.

General Feedback:
The correct response is : Atrial fibrillation with rapid ventricular response.
The ECG shows an irregularly irregular rhythm with an absence of P waves. It is considered to be “rapid”
atrial fibrillation when the ventricular rate exceeds 100 beats per minute.
Reference:
Life in the Fast Lane. Atrial fibrillation. Updated March 2021. https://litfl.com/atrial-fibrillation-ecg-library/
Image source:
Life in the Fast Lane. Atrial fibrillation. Updated March 2021. Example 4. https://litfl.com/atrial-fibrillation-ecg-l
ibrary/

Q.20
Mark Lowe, a 66 year old male, attended your clinic with his wife for his annual diabetes check. Mark has well-controlled insulin-dependent type 2
diabetes mellitus. Your practice recently purchased an ankle:brachial index (ABI), which shows an ABI for Mark of 1.2. His wife, Pam, 58, who is
otherwise well with no significant past medical history and no history of smoking, asks if she could book in for an ABI to screen for peripheral
vascular disease too.

What is the most appropriate advice to give Pam regarding this investigation? Choose one (1) option.
Pam would benefit from screening for peripheral vascular disease as she is over 50 years of age

Advise Pam that she should have a combined doppler venous ultrasound and ankle:brachial index (ABI) to assess
for peripheral vascular disease

Advise Pam that there is insufficient evidence to advocate screening in low risk patients and that she does not
require ankle:brachial index (ABI) testing

Pam should undergo a physical examination, with a focus on the absence of pulses, femoral bruit and trophic skin
changes as this is more sensitive than ankle:brachial index (ABI) testing.

Advise Pam that there no evidence for ankle:brachial index (ABI) screening in any population groups

Your answer is correct.

General Feedback:
The correct response is: Advise Pam that there is insufficient evidence to advocate screening in low risk
patients and that she does not require ankle:brachial index (ABI) testing
Current RACGP guidelines for preventive activities in general practice (the ‘Red Book’) does not advocate
for peripheral vascular disease screening with ankle:brachial index (ABI) in low risk patients. You can find
this in the section about screening tests of unproven benefit.
References:
The Royal Australian College of General Practitioners. Guidelines for preventive activities in general
practice. 9th ed, updated. East Melbourne, Vic: RACGP, 2018. Available from: https://www.racgp.org.au/clinic
al-resources/clinical-guidelines/key-racgp-guidelines/view-all-racgp-guidelines/guidelines-for-preventive-acti
vities-in-general-pr/screening-tests-of-unproven-benefit (Accessed December 2022).

Q 21
Q.21
Your patient, 73 year old Rohan Balci, is concerned about a new small pigmented skin lesion on his left forearm. Your examination confirms a 4mm
pigmented lesion with some border irregularity. You plan to perform a biopsy of the lesion but are concerned about bleeding risks as Rohan is on
aspirin 100mg orally daily and prasugrel 10mg orally daily which was commenced nine months ago following his ST elevation myocardial infarction.
This was treated with percutaneous coronary intervention and a single coronary artery stent.
What is the most appropriate way in which to manage Rohan’s skin lesion? Choose one (1) option.

Perform an excision biopsy under local anaesthetic with 2mm margins; cease aspirin and prasugrel the day prior to
the procedure and resume the next day following the excision.

Perform an excision biopsy of 4mm margins under local anaesthetic; cease aspirin and prasugrel for 10 days prior
to the procedure and resume the next day following the excision

Wait for three months until the planned cessation of prasugrel; then perform an excision biopsy with 4mm margins
under local anaesthetic, whilst continuing aspirin

Continue his current medications and perform an excision biopsy with 2mm margins under local anaesthetic

Continue his current medications and perform a punch biopsy of 3mm under local anaesthetic

Your answer is correct.

General Feedback:
Explanation:
The correct response is: Continue his current medications and perform an excision biopsy with 2mm
margins under local anaesthetic
A new pigmented skin lesion with border irregularity requires a histological diagnosis to exclude melanoma.
It is not appropriate to wait three months to obtain a histopathological diagnosis. Any cessation of dual
anticoagulant therapy in this patient would carry a significant risk of stent thrombosis and recurrent
myocardial infarction. There is likely to be increased bleeding at excision when on dual anticoagulation
therapy but an excision on the forearm might be managed with additional local measures such as extra local
pressure and additional sutures.
In addition, the margins of an excision biopsu should be 2mm for a suspicious pigmented lesion. A punch
biopsy of 4mm would not be suitable, given the lesion itself measures 4mm as this does not provide
adequate margins.
References:
Jayasinghe R, Markham R, Adsett G. Dual antiplatelet therapy: Management in general practice. Australian
Family Physician 2013; 42 (10): 702-705. https://www.racgp.org.au/afp/2013/october/dual-antiplatelet-therap
y/ (Accessed July 2022).

Chan J, Wood BA, Harvey NT. Skin biopsy in the diagnosis of neoplastic skin disease. Aust Fam Physician
[Internet]. 2017 May [cited 2022 Jul 18];46(5). Available from: https://www.racgp.org.au/afp/2017/may/skin-bio
psy-in-the-diagnosis-of-neoplastic-skin-di

Q.22
Franky Kngwarreye is a 37 year old Indigenous male who has attended today for a health check which was motivated by his 43 year old friend from his
football team recently requiring a coronary artery stent. Franky current has no symptoms of concern including no chest pain, shortness of breath,
palpitations or ankle swelling. He has smoked 20 cigarettes each day for the past 20 years but does not drink alcohol. He plays football three times
weekly and has a high level of physical fitness. Franky has a healthy diet as he likes to keep in good condition for his football competition games. He
is not aware of any family history of heart disease.

His blood pressure today is 135/85mmHg and his body mass index is 22kg/m2. You advise Franky to have fasting blood tests performed and his total
cholesterol (4.0 mmol/L) and HDL cholesterol (1.5 mmol/L) and renal function testing are within normal limits. Franky's urine albumin creatinine ratio is
within normal limits. You discuss smoking cessation with Franky today and he agrees to a trial of varenicline and counselling through Quitline.

What other pharmacological management advice would you give to Franky today in relation to management of his cardiovascular risk? Choose one
(1) option.

Recommend that he commences aspirin 100mg orally daily

Advise him that medication is not indicated at this time for primary prevention of cardiovascular disease but he
should return in 2 years’ time for review of his level of risk

Recommend that he commences perindopril 5mg orally daily and atorvastatin 40mg orally nocte
Recommend that he commences aspirin 100mg orally daily and perindopril 5mg orally daily

Recommend that he commences aspirin 100mg orally daily, perindopril 5mg orally daily and atorvastatin 40mg
orally daily

Your answer is correct.

General Feedback:
The correct response is: Advise him that medication is not indicated at this time for primary prevention of
cardiovascular disease but he should return in 2 years' time for review of his level of risk
Current Australian guidelines indicate the following:
"In Aboriginal and Torres Strait Islander adults aged 35-74 years who are not known to have
CVD or to be at clinically determined high risk, absolute cardiovascular risk over the next five years
should be calculated using the Framingham Risk Equation. Although the Framingham Risk Equation
might underestimate risk in this population, available evidence suggests that this approach will
provide an estimate of minimum cardiovascular risk."
Using the information provided, Franky's cardiovascular disease risk calculation result is 2%. This means he
has a 2% risk of experiencing a myocardial infarction or cerebrovascular accident within the next 5 years
which is considered "low risk" and aspirin, antihypertensives and lipid-lowering therapy would not be
indicated for him at this stage. However, given his Indigenous ethnicity, it is important to keep in mind that
this risk calculation may underestimate his true risk and regular review would be important.
Reference:
Heart Foundation. Absolute CVD risk clinical guidelines 2012. Available from: https://www.heartfoundation.or
g.au/conditions/fp-absolute-cvd-risk-clinical-guidelines (Accessed March 2022).
Heart Foundation. Absolute cardiovascular disease risk calculator. Available from: https://www.heartfoundati
on.org.au/health-professional-tools/cvd-risk-calculator (Accessed March 2022).

Q.23
Margaret Banks is a 49-year-old woman well known to your practice. She presents for her regular prescriptions. She is currently on telmisartan,
hydrochlorothiazide and amlodipine. Her blood pressure today is 150/100mmHg. You note that she has had similar blood pressure readings on three
recent separate occasions.
Which of the following criteria is consistent with the current screening recommendations for primary aldosteronism as a cause for Margaret’s
hypertension? Choose one (1) option.

Hypertension with spontaneous or diuretic-induced hyperkalaemia

Hypertension (BP >140/90mmHg) resistant to three conventional anti-hypertensive drugs (including a diuretic)

Hypertension and a family history of hypertension or cerebrovascular accident at less than 60 years of age

Hypertension presenting with a single systolic blood pressure reading above 180mmHg

Hypertension (BP >170/100mmHg) with proteinuria

Your answer is correct.

General Feedback:
The correct response is: Hypertension (BP >140/90mmHg) resistant to three conventional anti-hypertensive
drugs (including a diuretic)
While primary aldosteronism accounts for 5% - 10% of hypertension in general practice and up to 30% of
those with refractory hypertension, the Bettering the Evaluation and Care of Health (BEACH) program
revealed that aldosterone measurement was only ordered on 66 occasions in over 1,500,000 GP-patient
encounters. Screening for cases of secondary hypertension is low relative to the number of patients with
hypertension who fulfil current screening recommendations.
The Endocrine Society's current screening recommendations for primary aldosteronism include:
Sustained blood pressure (BP) above 150/100 mmHg on each of three measurements obtained on
different days, or
Hypertension (BP >140/90 mmHg) resistant to three conventional antihypertensive drugs (including a
diuretic), or
Controlled BP (<140/90 mmHg) on four or more antihypertensive medications
Hypertension and spontaneous or diuretic-induced hypokalaemia
Hypertension and adrenal incidentaloma
Hypertension and sleep apnoea
Hypertension and a family history of early-onset hypertension or cerebrovascular accident at a young
age (<40 years)
All hypertensive first-degree relatives of a patient with primary aldosteronism
References:
Gurgenci T, Geraghty S, Wolley M, Yang J. Screening for primary aldosteronism: How to adjust existing anti-
hypertensive medications to avoid diagnostic errors. Aust Journal of General Practice. 2020; 49 (3): 127-131.
Available from: https://www1.racgp.org.au/ajgp/2020/march/screening-for-primary-aldosteronism
Lim YY, Shen J, Fuller P et al. Current pattern of primary aldosteronism diagnosis. Aust Journal of General
Practice 2018; 47: 712-718. Available from: https://www1.racgp.org.au/AJGP/2018/October/Current-pattern-of-
primary-aldosteronism-diagnosis

Q.24
Bill Cook is a 72-year-old male who presents with fatigue. He has a past medical history of an acute myocardial infarction at age 55. He states that over
the last 6 months he has become more tired with normal daily activities. He lives in a 2-storey house and gets breathless when climbing the stairs. He
denies any symptoms at rest and can dress himself without feeling short of breath. He states that he has put on 3 kg over the last 3 months and his
socks leave quite distinctive marks around his ankles. You suspect that Bill has chronic heart failure.
What class of the New York Heart Association (NYHA) functional classification system of chronic heart failure symptoms would you assign to Bill?
Choose one (1) option.

NYHA Class 1

NYHA Class 2

NYHA Class 3

NYHA Class 4

NYHA Class 5
Your answer is incorrect.

General Feedback:
The correct response is: NYHA Class 2
NYHA Class 1 indicates no limitation of physical activity. Class 2 indicates slight limitation of ordinary
physical activity, with no symptoms at rest. Class 3 indicates marked limitation of physical activity with no
symptoms at rest. Class 4 is defined as symptoms on any physical activity or at rest.
The patient in this clinical stem is breathless with climbing stairs, but can perform his other activities of daily
living without symptoms. As such, Class 2 is the best fit.

Reference:
National Heart Foundation of Australia and Cardiac Society of Australian and New Zealand: Guidelines for
the Prevention, Detection, and Management of Heart Failure in Australia 2018. Available from:
https://www.heartlungcirc.org/article/S1443-9506(18)31777-3/fulltext#sec0165 (See section 3.2; table 2)
(Accessed December 2022)

Correct answers is: NYHA Class 2

Q.25
Gwen Sorcen, a 75 year-old woman, presents complaining of intermittent palpitations with fatigue and shortness of breath over the last few months.
She has a past history of hypertension and diabetes. You perform an electrocardiogram which confirms atrial fibrillation.

Based on the information provided, what is Gwen’s CHA2 DS2-VA score? Choose one (1) option.

3
4

Your answer is correct.

General Feedback:
The correct response is: 4
The 2018 Australian Clinical Guidelines for the Diagnosis and Management of Atrial Fibrillation recommends
the use of the CHA2 DS2-VA score to assess stroke risk. Gwen’s score is 4 which was calculated as follows:
History of hypertension: 1 point
Age ≥75 years: 2 points
Diabetes: 1 point
ScorePointsDefinition
Congestive heart failure—recent signs, symptoms or admission for decompensated heart failure; this
C 1 includes both HFrEF and HFpEF, or moderately to severely reduced systolic left ventricular function,
whether or not there is a history of heart failure
H 1 History of Hypertension, whether or not BP is currently elevated
A2 2 Age ≥75 years
D 1 Diabetes
S2 2 History of prior Stroke or TIA or systemic thromboembolism
Vascular disease, defined as prior myocardial infarction or peripheral arterial disease or complex aortic
V 1
atheroma or plaque on imaging (if performed)
A 1 Age 65–74 years

An updated version of the scoring system for assessing stroke risk is the CHA2 DS2-VASc score, with 'Sc'
referring to the sex category. On its own or with only one additional risk factor, female sex is not associated
with an increased risk of stroke in the setting of atrial fibrillation. With additional risk factors, female sex
adds an additional point to the score. When the female sex category is added, a different scoring system is
then used for male and female patients. To avoid this practice, the guidelines below recommends a sexless
CHA2DS2-VASc score (i.e. removing female sex), abbreviated as CHA2DS2-VA score, with one consistent
recommendation for both sexes.
Reference:
National Heart Foundation of Australia and the Cardiac Society of Australia and New Zealand. Australian
Clinical Guidelines for the Diagnosis and Management of Atrial Fibrillation 2018. https://www.heartlungcirc.or
g/article/S1443-9506(18)31778-5/fulltext (Accessed July 2022).

Q.26
Betty Smith is a 70 year old female who has newly diagnosed paroxysmal non-valvular atrial fibrillation. She has a history of diabetes and is on
perindopril for hypertension. She has no history of falls, stroke / transient ischaemic attack, thromboembolism, heart failure, vascular disease, renal or
liver disease and lives independently in her own home. Her weight is 65 kg.

When considering stroke prevention for this patient using her CHA2DS2-VASc score, which one of the following is most appropriate next step in
management? Choose one (1) option.

Begin aspirin 100mg orally daily

Begin apibaxan 5mg orally twice daily

No treatment is required to reduce her stroke risk

Commence Betty on oral warfarin, aiming for an international normalised ratio of 2-3

Refer Betty for a pacemaker insertion

Your answer is correct.

General Feedback:
The correct response is: Begin apibaxan 5mg orally twice daily
The 2018 Australian Clinical Guidelines for the Diagnosis and Management of Atrial Fibrillation recommends
the use of the CHA2 DS2-VASc score to assess stroke risk. This patient’s score is 4:
History of hypertension: 1 point
Age 65-74 years: 1 point
Diabetes: 1 point
Female gender: 1 point
In patients with non-valvular AF and with no contraindications to anticoagulation, the guidelines
recommend:
for men with a CHA2DS2-VASc score of 2 or more and women with a score of 3 or more, strong
evidence shows benefit of oral anticoagulant therapy. Give oral anticoagulant therapy to these
patients
for men with a CHA2DS2-VASc score of 1 and women with a score of 2, evidence for benefit of oral
anticoagulant therapy is less strong. Consider oral anticoagulant therapy for these patients
for men with a CHA2DS2-VASc score of 0 and women with a score of 1, anticoagulant therapy is not
recommended.
Bleeding risk also needs to be taken into consideration when making a decision about anticoagulation.

When anticoagulation is recommended in a patient with non-valvular atrial fibrillation, a Non–vitamin K


antagonist oral anticoagulants (NOACs) (apibaxan, dabigatran or rivaroxaban) is recommended in
preference to warfarin, provided there are no contraindications. In patients with valvular AF requiring
anticoagulation, warfarin should be used rather than NOACs. There is no role for the use of anti-platelet
therapy for stroke prevention in patients with non-valvular atrial fibrillation. We are told that Betty has
normal renal and liver function so dose adjustments of apixaban are not needed.

Reference:
Therapeutic Guidelines. Atrial Fibrillation [Internet]. Available from: https://tgldcdp.tg.org.au/viewTopic?etgA
ccess=true&guidelinePage=Cardiovascular&topicfile=atrial-fibrillation&guidelinename=Cardiovascular&secti
onId=toc_d1e149#toc_d1e149 (Accessed July 2022)
National Heart Foundation of Australia and the Cardiac Society of Australia and New Zealand. Australian
Clinical Guidelines for the Diagnosis and Management of Atrial Fibrillation 2018. https://www.heartlungcirc.or
g/article/S1443-9506(18)31778-5/fulltext (Accessed July 2022).

Q.27
Kasun Chandran is a 58 year old man with type 2 diabetes for which he takes metformin 1000mg orally daily. Kasun is a regular patient of yours and
you commenced him on atorvastatin 10mg orally nocte four weeks ago due to dyslipidaemia after assessing that his cardiovascular risk was high.
During that appointment, you warned Kasun of the rare side effects of statins to look out for. In addition to your advice about medication, you
encouraged Kasun to increase his physical activity in order to improve his cardiovascular risk. Kasun reported that he planned to buy a gym
membership in order to keep himself motivated.

Kasun has come to see you today, concerned that he may be experiencing statin-associated myopathy. He reports that his urine still appears a normal
colour.

Which of the following symptoms would be most concerning for statin-associated muscle symptoms in Kasun? Choose one (1) option.
Muscle soreness in both of his shoulders

Pain in both of his knees when walking

Intermittent twitching of the right side of his face

Muscle aching in his right leg

The presence of cramps in his calves at night when he is in bed

Your answer is correct.

General Feedback:
The correct answer is: Muscle soreness in both of his shoulders
Muscle-related symptoms are a common complaint for many patients, including those on statins. Many of
these symptoms are found not to be statin-associated muscle symptoms (SAMS) and may be due to other
causes. Features which are more suggestive of SAMS include:
Bilateral distribution
Large muscle groups, e.g. thighs, buttocks, shoulders, calves
The discomfort may be described as an ache, weakness, soreness, stiffness, cramping, tenderness and
general fatigue.
Onset 4-6 weeks after statin initiation or dose increase
SAMS is more likely in people who engage in vigorous physical activity. In Kasun’s case, the presence of
muscle soreness in his shoulders is most suggestive of SAMS, out of all the available options. Going on to
assess his creatine kinase level is the next step.
SAMS is less likely in people who have the following presentation including:
Unilateral distribution
Tingling, twitching, shooting pain, nocturnal cramps or joint pain
Onset >12 weeks after statin initiation
References:
NPS Medicine Wise. Statin-associated muscle symptoms [Internet]. 2017 Jun 19 [cited 2021 Sep 13].
Available from: https://www.nps.org.au/professionals/managing-lipids/statin-associated-muscle-symptoms-s
ams

Q.28
You have been seeing John, a 65 year old patient for many years. In the last 5 years he has developed progressively worsening Chronic Kidney
Disease, but with tighter blood pressure control you have managed to keep his estimated Glomerular Filtration Rate (eGFR ) stable for the last 18
months at 50mL/min/1.73m2 (Normal range >90mL/min/1.73m2) with normoalbuminuria.

In regards to patients with Chronic Kidney Disease, which of the following is the recommended target levels for blood pressure control? Choose one
(1) option.

≤130/80mmHg

≤150/90mmHg

≤130/90mmHg

≤140/90mmHg

≤120/80mmHg

Your answer is correct.

General Feedback:
The correct response is: ≤130/80
All people with diagnosed CKD should maintain a blood pressure ≤130/80.
Hypertension has been recognised as both a cause of Chronic Kidney Disease and a complication. Poorly
controlled blood pressure can contribute to progression of Chronic Kidney Disease. See the Australian
guidelines referenced below (page 45) for details about hypertension management in patients with CKD.
Aiming for a systolic blood pressure of <120 mmHg may be appropriate in certain individuals who are at very
high cardiovascular risk.
Lower blood pressure targets needs to be balanced with an increased risk of side effects including
hypotension leading to increased falls, syncope, electrolyte abnormalities and increased episodes of AKI.
Reference:
Chronic Kidney Disease Management in Primary Care; 4th Edition, 2020. https://kidney.org.au/health-professi
onals/prevent/chronic-kidney-disease-management-handbook (Accessed November 2022)

Q.29
Phyllis MacDougall is a 69-year-old woman who lives at home with her husband. She is a long-standing patient of yours and you see her monthly for
monitoring of her International Normalised Ratio (INR). Phyllis has a history of atrial fibrillation, mitral stenosis, hypertension, and osteoarthritis.
Phyllis' INR has been within the target range of 2-3 for the last 6 months and her dose of warfarin (Marevan) has remained at 4mg orally daily. Her
other regular medications include enalapril/hydrochlorothiazide 20/6mg orally daily and paracetamol 1330mg orally twice a day.

Phyllis last had her renal and liver function tests checked two months ago. Her eGFR was 69 mL/min/1.73m2 and her liver function tests were within
the normal range.

Today, Phyllis' INR is 3.8. She assures you that she has been taking her warfarin as prescribed and that she has not changed brands or changed her
diet significantly. Phyllis tells you that she had a nosebleed this morning which terminated after applying local pressure for 20 minutes. She denies
any blood in her stools or any headaches or abdominal pain. Her weight is 70 kg. Phyllis appears alert and orientated. She denies any falls over the
last 12 months.

Which of the following is the next most appropriate course of action? Choose one (1) option.

Advise Phyllis not to take her warfarin today and then to commence it at 3mg tomorrow and recheck her INR the
following day

Advise Phyllis not to take her warfarin today and then recheck her INR tomorrow

Give Phyllis a single dose of 1mg Vitamin K orally and ask her to omit her warfarin dose today, before restarting it
at 4mg tomorrow
Cease warfarin immediately and commence apixaban 5mg orally once her INR is <2.5

Refer Phyllis to the emergency department for urgent administration of prothrombin complex

Your answer is correct.

General Feedback:
The correct answer is: Advise Phyllis not to take her warfarin today and then recheck her INR tomorrow
Phyllis has valvular atrial fibrillation (AF), defined as AF in the setting of an artificial heart valve or mitral
stenosis. Direct-acting anticoagulants such as apixaban are not indicated for valvular AF so swapping to
apixaban is not an option for Phyllis.
When assessing a patient for possible over-anticoagulation, it is important to establish if they have any
bleeding and whether this is minor (such as a nosebleed) or major (such as a gastrointestinal or intracranial
bleed). If minor bleeding is present and the INR is <4.5, holding the next dose of warfarin and rechecking the
INR the following day is the most appropriate course of action. If there is further over-
anticoagulation/bleeding it may be appropriate to continue to hold warfarin the following day as well.
Recommencing warfarin without an updated INR level first is not recommended.
Major bleeding is an indication for emergency department referral and possible commencement of
prothrombin complex or even fresh frozen plasma. For patients with minor bleeding or who are at risk of
bleeding and an INR >4.5, a single dose of 1-2mg of Vitamin K orally may be appropriate. In this case, Phyllis’
INR is 3.8 and she therefore does not warrant Vitamin K.
Reference:
Australian Medicines Handbook. Warfarin [Internet]. 2022 Jan [cited 2022 Feb 28]. Available from: https://amh
online.amh.net.au/chapters/blood-electrolytes/other-drugs-affecting-haemostasis/vitamin-k#vitamin-k-dosag
e-01 (paid subscription required)
Tran HA, Chunilal SD, Harper PL, Tran H, Wood EM, Gallus AS. An update on consensus guidelines on
warfarin reversal. Med J Aust [Internet]. 2013 [2022 Feb 28];198(4) Available from: https://www.mja.com.au/jo
urnal/2013/198/4/update-consensus-guidelines-warfarin-reversal

Q.30
Bill Cook is a 72-year-old male who presents with fatigue. He has a past medical history of an acute myocardial infarction at age 55. He states that over
the last 6 months he has become more tired with normal daily activities. He lives in a 2-storey house and gets breathless when climbing the stairs. He
denies any symptoms at rest and can dress himself without feeling short of breath. He states that he has put on 3 kg over the last 3 months and his
socks leave quite distinctive marks around his ankles. You arrange an echocardiogram, which demonstrates impaired left ventricular contractile
function (LVEF < 40%).

Which of the following statements is correct? Choose one (1) option.

Loop diuretics are proven to improve survival outcomes in patients with heart failure with reduced ejection fraction

All patients with heart failure with reduced ejection fraction should be prescribed a nitrate patch

Beta blockers may initially worsen symptoms in patients with heart failure with reduced ejection fraction

Aldosterone antagonists have not been shown to reduce hospitalisation rates in patients with heart failure with
reduced ejection fraction

Patients with heart failure with reduced ejection fraction should be encouraged to avoid physical activity

Your answer is correct.

General Feedback:
The correct response is: Beta blockers may initially worsen symptoms in patients with heart failure with
reduced ejection fraction
Beta blockers can also cause severe hypotension and bradyarrhymias. Therefore it is recommended that
beta blockers are started at a very low dose and increased very gradually in patients with HFrEF. They
should not be commenced during a period of acute decompensation. The patient should be reviewed
regularly and advised to perform daily weighs. It is recommended that vasodilator drugs are not commenced
at the same time as beta blockers.
Reference:
eTG complete. Heart failure. Published 2018, amended 2022. Available from:
https://tgldcdp.tg.org.au/viewTopic?topicfile=heart-failure (Accessed December 2022; requires paid
subscription)
Q.31
Mark Brookes is a 49 year old man who presents for a routine check-up. He reports no significant past medical history and takes no regular
medications. On examination, his blood pressure is 210/110mmHg. You keep Mark in the clinic and perform serial BPs in the treatment room which
do not improve significantly. He is asymptomatic from his hypertension.

On further questioning Mark thinks his blood pressure was high a few years ago, but he moved towns and forgot to follow it up. You consider if he
may have secondary hypertension. Which of the following statements about secondary hypertension in adults is correct? Choose one (1) option.

Secondary hypertension is more common in those with a family history of hypertension

The most common causes of secondary hypertension in Mark's age group include: renal parenchymal disease and
coarctation of the aorta

The most common causes of secondary hypertension in Mark's age group include: renal artery stenosis, and renal
failure

The most common causes of secondary hypertension in Mark's age group include: obstructive sleep apnoea and
adrenal/endocrine disorders.

Management of the underlying cause will not improve secondary hypertension

Your answer is correct.

General Feedback:
The correct response is: The most common causes of secondary hypertension in Mark's age group include:
obstructive sleep apnoea and adrenal/endocrine disorders.
This includes conditions such as: Hyperaldosteronism, thyroid dysfunction, Cushing's syndrome and
pheochromocytoma.
Aetiologies of secondary hypertension are different in children compared to adults. In children (birth-11
years) renal parenchymal disease (eg reflux nephropathy, glomerulonephritis, polycystic kidney disease)
and coarctation of the aorta are the most common causes of secondary hypertension. In older adults (65
years and over) atherosclerotic renal artery stenosis and renal failure are the most common causes.
Secondary hypertension is more common in patients without a family history of hypertension. Management
of the underlying cause can improve or normalise blood pressure in patients with secondary hypertension.
Reference:
eTG. Secondary hypertension. https://tgldcdp.tg.org.au/viewTopic?topicfile=blood-pressure-reduction#toc_d
1e617 (Accessed June 2020).
Charles L, Triscott J and Dobbs B. Secondary Hypertension: discovering the underlying cause. American
Family Physician 2017; 96(7):453-461
https://www.aafp.org/afp/2017/1001/p453.html (Accessed June 2020)

Q.32
Mrs Miriam White is a 72 year old regular patient who is usually well and lives independently with her husband. She presents today with a sudden
onset of a ‘fluttering’ in her chest. She reports feeling a bit breathless and denies any chest pain. She has a past history of hypertension (for which
she takes ramipril 5mg orally daily) and mitral valve prolapse. Your practice nurse has seen her in the waiting room and taken her straight to the
treatment room for an ECG.
What rhythm is shown on Miriam’s ECG trace? Choose one (1) option.
Atrial flutter

Atrial flutter with 2:1 block

Wolff Parkinson White Syndrome

Rapid atrial fibrillation

Sinus tachycardia with peaked QRS complexes suggestive of hyperkalaemia


Your answer is correct.

General Feedback:
The correct response is: Rapid atrial fibrillation
This ECG shows an irregular narrow complex tachycardia with a rate of 135bpm with loss of P waves and an
irregular ventricular response.
Atrial flutter has regular atrial activity and typically produces a rate of 300bpm, or 150bpm in flutter with 2:1
block. Wolff Parkinson White Syndrome usually exhibits a rate >200bpm with wide QRS complexes due to
ventricular depolarization in the accessory pathway. Hyperkalaemia usually produces peaked T waves (not
peaked QRS complexes). It then progressively causes flattening of the P waves and a prolonged QRS
interval.

Reference:
Nickson, C. Atrial Fibrillation. Life in the Fast Lane. Updated December 2021. Available from: https://litfl.com/
atrial-fibrillation-ecg-library/ (Accessed August 2022).

Image reference:
Life in the Fast Lane. ECG library: Atrial Fibrillation. Updated December 2021. Available from: https://litfl.com/
atrial-fibrillation-ecg-library/ (Accessed August 2022).

Q.33
Patricia is a 65 year old female patient who you have been seeing for several years for management of her diet-controlled type 2 diabetes. She reports
a healthy diet which is low in salt and she has recently lost 10% of her body weight. Over the past six months she has started walking for 60 minutes
each day with a friend. Patricia is a non-smoker and non-drinker. Six months ago you diagnosed her with hypertension and prescribed perindopril
arginine 5mg daily which she has been taking regularly with no side effects.

During Patricia’s last consultation three months ago you noted that her blood pressure was still 168/95mmHg. She had some blood pressure readings
performed at the local pharmacy since then which show similar readings. You recheck her blood again today and it is 170/90mmHg.

How do you manage Patricia’s blood pressure now? Choose one (1) of the following options:
Advise Patricia that no change to her anti-hypertensive medication is required at this stage and encourage
continued lifestyle change

Add amlodipine 5mg daily

Add metoprolol 50mg daily

Increase Perindopril arginine to 10mg daily

Add Irbesartan 150mg daily

Your answer is incorrect.

General Feedback:
The correct response is: Add amlodipine 5mg daily
In patients with diabetes and hypertension, any of the first-line antihypertensive drugs that effectively lower
blood pressure are recommended. In patients with uncomplicated hypertension ACE inhibitors or ARBs,
calcium channel blockers, and thiazide diuretics are all suitable first-line antihypertensive drugs.
If monotherapy with a first line drug does not sufficiently reduce blood pressure the next step is to add a
second first line- anti hypertensive medication - in this case either a calcium channel blocker or a low dose
thiazide or thiazide like diuretic.
Combination therapy, defined by the use of at least two antihypertensive drugs, is required in up to 50–70%
of patients to reach blood pressure targets. It is widely accepted that combining two classes of
antihypertensive drug lowers blood pressure more than doubling the dose of one drug and may have a lower
risk of side effects.
The balance between efficacy and safety is less favourable for beta-blockers than other first-line
antihypertensive drugs. Thus beta-blockers should not be offered as a first-line drug therapy for patients
with hypertension not complicated by other conditions.
ACE inhibitors and ARBs are not recommended in combination due to the increased risk of adverse effects.
References:
The Royal Australian College of General Practitioners. Management of Type 2 Diabetes: A Handbook for
General Practice. 2020 Jul. https://www.racgp.org.au/getattachment/41fee8dc-7f97-4f87-9d90-b7af337af778/M
anagement-of-type-2-diabetes-A-handbook-for-general-practice.aspx (Accessed November 2022)
Heart Foundation. Guideline for the diagnosis and management of hypertension in adults. 2016. Available
from: https://www.heartfoundation.org.au/getmedia/c83511ab-835a-4fcf-96f5-88d770582ddc/PRO-167_Hypert
ension-guideline-2016_WEB.pdf (Accessed November 2022)
Correct answers is: Add amlodipine 5mg daily

Q.34
Elliott Doust is a 49 year old man who has presented for his 45-49 year old health assessment. He tells you that he continues to smoke twenty
cigarettes a day. Elliot has his first cigarette with his morning coffee shortly after he wakes up.

Elliott has a history of epilepsy for which he takes carbamazepine controlled release 200mg orally twice a day and sodium valproate 500mg orally
twice a day. His last tonic-clonic seizure was 7 months ago.

You assess his readiness to quit smoking and Elliott tells you that he has always wanted to quit but feels he would need support to achieve this. You
recommend that he contact Quitline for support and counselling and suggest pharmacotherapy. Elliott tells you that he is interested in the most
effective option that is suitable for him.

Which of the following options is the most appropriate smoking cessation pharmacotherapy for Elliott? Choose one (1) option.

Nicotine 21mg/24 hour patch daily with Nicotine lozenge 2mg as needed

Varenicline 0.5mg orally daily, increasing to 1mg twice daily

Nortriptyline 10mg orally daily

Bupropion 150mg orally daily, increasing to 150mg twice daily

Nicotine 14mg/24 hour patch daily with Nicotine lozenge 2mg as needed

Your answer is correct.


General Feedback:
The correct response is: Nicotine 21mg/24 hour patch daily with Nicotine lozenge 2mg as needed
Elliott has displayed evidence of nicotine dependence, with his first cigarette within 30 minutes of waking
up. In addition, he smokes more than 10 cigarettes a day which is also a sign of nicotine dependence. Both
varenicline (Champix) and combined Nicotine Replacement Therapy (NRT) are equally effective at
encouraging smoking cessation. Varenicline should be used with caution in people with a history of epilepsy
as it may provoke seizures. Therefore, in Elliott’s case, combined NRT would be a safer alternative.
People who smoke more than 10 cigarettes per day require a patch strength of 21mg/24 hours or 15mg/16
hours. For people who smoke 10 cigarettes or less per day, a Nicotine patch strength of 14mg/24 hours or
10mg/16 hours is recommended.
Bupropion is contraindicated in people with a history of seizures. Nortriptyline has some evidence for
improving smoking cessation rates however the evidence is not as robust. Nortriptyline can also cause
cholinergic side effects which may not be well tolerated.
References:
Zwar NA. Smoking Cessation. Aust J Gen Pract[Internet]. 2020 Aug. Available from: https://www1.racgp.org.a
u/ajgp/2020/august/smoking-cessation-1 (Accessed July 2021).
Therapeutic Guidelines. Smoking Cessation [Internet]. 2013 Jul. Available from: https://tgldcdp.tg.org.au/vie
wTopic?topicfile=smoking-cessation (Accessed July 2021).

Q.35
Peter, a 62 year old man presents for a check-up at his wife's insistence as her 65 year old brother has recently had a myocardial infarction. He doesn't
really want to come for review and asks you if you can tell him today if he is at an increased risk of cardiovascular disease. He had some blood tests
performed a year ago for an insurance medical.

In deciding if he needs further assessment, which one of the following automatically puts Peter at high risk of cardiovascular disease? Choose one (1)
option.

Diastolic blood pressure 100 mmHg

Age > 60 years

eGFR 65 mL/min/1.73m2
Systolic blood pressure 180 mmHg

Serum total cholesterol 7.2 mmol/L

Your answer is incorrect.

General Feedback:
The correct response is: Systolic blood pressure 180 mmHg
Patients with a systolic blood pressure of equal to or greater than 180mmHg are considered at high risk. See
the Heart Foundation's guide to absolute cardiovascular disease risk assessment for a full list of criteria
which makes a patient at high risk.
Reference:
National Vascular Disease Prevention Alliance. Guidelines for the management of absolute cardiovascular
risk. 2012. Available from: https://www.heartfoundation.org.au/getmedia/4342a70f-4487-496e-bbb0-dae33a47f
cb2/Absolute-CVD-Risk-Full-Guidelines_2.pdf (Accessed May 2020).
Correct answers is: Systolic blood pressure 180 mmHg

Q.36
Jaimie Ellis is a 26 year old woman who presents to your rural emergency department with palpitations which came on suddenly about an hour ago.
She has had no associated chest pain, breathlessness or light-headedness. She has had no previous episodes of palpitations and is usually well and
takes no regular medications. Jaimie denies any prior history of cardiac problems. She does not smoke, drinks alcohol occasionally and does not use
recreational drugs. She has no significant family history.

On examination, she is alert, with a respiratory rate 18 breaths / minute, SpO2 100% on room air, heart rate of 150 beats / minute and a blood pressure
of 118/74mmHg. Her electrocardiogram is shown below:
Which is the following is the most appropriate first-line management for Jaimie? Choose one (1) option.

Arrange urgent transfer to the closest tertiary hospital

Digoxin 250 micrograms orally stat

Metoprolol 25mg orally 12 hourly

Modified Valsalva manoeuvre

Adenosine 6mg intravenously STAT

Your answer is correct.

General Feedback:
The correct response is: Modified Valsalva manoeuvre
This electrocardiogram (ECG) demonstrates supraventricular tachycardia (SVT) in a patient who is otherwise
stable.
Immediate transfer to a tertiary centre is not indicated at this stage as she is stable and you have capacity to
trial first-line management in the rural ED.
Narrow-complex SVT can be converted to sinus rhythm by manoeuvres that enhance vagal tone. These
include the Valsalva manoeuvre and carotid sinus massage. The standard Valsalva manoeuvre is
recommended internationally as one of several treatment options for patients with SVT. In clinical practice,
however, it terminates SVT in only 5–20% of cases.
In the largest randomised trial of vagal manoeuvres for the treatment of SVT, the modified Valsalva was
shown to revert 43% of participants with SVT to sinus rhythm at one minute, with a number needed to treat
(NNT) of 4 to restore sinus rhythm in one patient. This trial did not identify any adverse events associated
with the modified Valsalva manoeuvre technique.
Carotid sinus massage is less effective than the Valsalva manoeuvre and has around 1% risk of neurological
complications, although these are usually transient.
If these manoeuvres are ineffective then one of the following drugs can be used with close blood pressure
monitoring as per Therapeutic Guidelines (eTG): Adenosine 6mg intravenously by rapid bolus, verapamil 5-
10mg over 2 minutes or metoprolol tartrate 2.5-5mg intravenously over 2 minutes.
Please click on the following link to see a video demonstrating the modified Valsalva manoeuvre for the
emergency treatment of supraventricular tachycardias (SVT). https://www.youtube.com/watch?v=8DIRiOA_O
sA
Image source:
Burns E and Buttner R. Supraventricular Tachycardia (SVT). Life in the Fastlane. 2021. Available from: https://
litfl.com/supraventricular-tachycardia-svt-ecg-library/ (Accessed March 2022).
Video source: The Lancet. Modified Valsalva manoeuvre for supraventricular tachycardia. 2015. Available
from: https://www.youtube.com/watch?v=8DIRiOA_OsA (Accessed March 2022).
Reference:
Therapeutic Guidelines (eTG). Acute management of paroxysmal supraventricular tachycardia. 2021.
Available from: https://tgldcdp.tg.org.au/viewTopic?topicfile=paroxysmal-supraventricular-tachycardia&guid
elineName=Cardiovascular&topicNavigation=navigateTopic#toc_d1e95 (Accessed March 2022).
RACGP. Handbook of Non-Drug Interventions (HANDI) interventions. Modified Valsalva manoeuvre for
supraventricular tachycardia. First Published 2016. Available from: https://www.racgp.org.au/clinical-resourc
es/clinical-guidelines/handi/conditions/cardiovascular/modified-valsalva-manoeuvre-for-supraventricular-
t (Accessed March 2022).

Q 37
Q.37
Rachel Anderson is a 58 year old female who is your regular patient. She presents to you for the results of her ‘heart health check’ blood tests. She is
generally very well; her past medical history includes tonsillectomy as a child and mild eczema. She takes no regular medications. Rachels mother
died suddenly from a heart attack in her late 50’s and Rachel is concerned about her risk given she is now at the same age that her mother was when
she died. This is what prompted her to come in and see you for a detailed heart check. Rachel tells you she is proud of how healthy she is for her age
and that she has worked hard to maintain a healthy lifestyle, has never smoked and does not drink alcohol. She currently feels very well.

Based on her results you calculate her Framingham Risk Score for Hard Coronary Heart disease as 7.8% 10 year risk of myocardial infarction or death.
You explain to her that that is considered ‘low’ risk for myocardial infarction over the next 10 years.
Despite this Rachel is still concerned and wants more ‘tests’ to assess her heart.

Which of the following investigations would be most appropriate to further investigate Rachels cardiovascular risk?

Please choose one (1) from the following options.

Exercise stress test

Echocardiogram

Coronary angiography

Coronary artery calcium score

Serum brain natriuretic peptide (BNP)

Your answer is incorrect.

General Feedback:
The correct response is: Coronary artery calcium score
Coronary artery calcium score is appropriate for asymptomatic patients at intermediate risk and for lower
risk patients (6-10% 10 year risk) with concerning features not considered by the Framingham Risk Equation.
This includes patients with a family history of premature coronary heart disease.
Very high risk and very low risk patients are not suitable for coronary artery calcium scoring. The above
table from the Australian Journal of General Practice article ‘Coronary artery calcium in primary prevention’
outlines patient suitability for calcium scoring.
The other listed options here are not appropriate for this patient as she is currently asymptomatic.
Reference:
Chua A, Blankstein R, Ko, B. Coronary artery calcium in primary prevention. AJGP 2020: 49(8). Available
from: https://www1.racgp.org.au/ajgp/2020/august/coronary-artery-calcium-in-primary-prevention (Accessed
August 2021).
Correct answers is: Coronary artery calcium score

Q.38
Tania is a 50 year old woman who underwent ablative therapy for the treatment of her AV nodal reentry tachycardia 3 months ago. She has had
excellent outcome and has ceased all her previous medications which included amiodarone. She presents today though feeling unwell. She describes
unintentional weight loss of 5kg in the last month, this is associated with a tremor, and a feeling of weakness. Today she feels like her heart is racing
again, and she is worried that the ablation has failed.

What is the MOST likely diagnosis? Choose one (1) from the following list:

Addison’s disease

Anxiety

Chronic Renal Failure

Congestive cardiac failure

Inflammatory pulmonary toxicity

Diabetes insipidus

Diabetes Mellitus

Recurrence of AV nodal re-entry tachycardia


Hepatic Failure

Hyperparathyroidism

Hyperthyroidism

Hypopituitarism

Hypothyroidism

Iron deficiency anaemia

Lymphoma

Multiple Myeloma

Psychosis

Your answer is correct.

General Feedback:
The correct answer is: Hyperthyroidism
This woman is presenting with Amiodarone induced thyrotoxicosis. Amiodarone-induced thyrotoxicosis can
develop during long-term amiodarone treatment, or some months after stopping treatment (because
amiodarone has a long half-life). It can occur in patients with and without pre-existing thyroid disease.
Amiodarone-induced thyrotoxicosis can be related to the high iodine content of amiodarone (type 1
amiodarone-induced thyrotoxicosis), but amiodarone can also directly damage thyroid cells (type 2
amiodarone-induced thyrotoxicosis).
Reference:
1. Tsang W and Houlden R. Amidarone- induced thyrotoxicosis: A review. Can J Cardiol. 2009 Jul; 25(7): 421–
424. Available from: https://www.ncbi.nlm.nih.gov/pmc/articles/PMC2723027/#:~:text=It%20typically%20occur
s%20in%20patients,eventual%20recovery%20in%20most%20patients.
2. Therapeutic Guidelines eTG complete. Bone and Metabolism: Thyrotoxicosis and hyperthyroidism:
Amiodarone induced thyrotoxicosis. 2019. Amended March 2022.
https://tgldcdp.tg.org.au/viewTopic?topicfile=thyrotoxicosis-and-hyperthyroidism§ionId=bmg2-c03-s14#bmg
2-c03-s14

Q.39
You are seeing Kath Jannes, a 62 year old female patient who has had consistently high blood pressure readings in the clinic over the past six
months. Today her blood pressure is 162/97mmHg. She reassures you she just has “white coat hypertension” and that she is not concerned about her
blood pressure.

You recommend that Kath performs home blood pressure readings using the clinic's automated machine which can be loaned out to patients. Kath is
not keen to perform a 24 hour ambulatory blood pressure recording. .

Which of the following statements about home blood pressure readings is correct? Choose one (1) option.

Blood pressure should be monitored at different times in the morning and evening

Blood pressure should be checked after eating and vigorous exercise

Blood pressure should be checked before taking regular daily medications

Blood pressure should be taken three times daily for four consecutive days

Caffeine and cigarettes should be avoided for at least 15 minutes before measuring blood pressure

Your answer is correct.

General Feedback:
The correct response is: Blood pressure should be checked before taking regular daily medications
Patients should be given advice on how to correctly take home blood pressure readings. They should be
advised to take the readings before medication, food and vigorous exercise for seven consecutive days. The
readings should be taken at the same time each morning and night. Cigarettes and caffeine need to be
avoided for at least 30 minutes before taking a reading.
Image source:
Sharman, J. et al. How to measure home blood pressure: recommendations for healthcare professionals and
patients. Australian Family Physician 2016; 45(1-2): 31-34.
https://www.racgp.org.au/afp/2016/januaryfebruary/how-to-measure-home-blood-pressure-recommendations
-for-healthcare-professionals-and-patients/

Reference:
Heart Foundation. Blood pressure and your heart. Available from: https://www.heartfoundation.org.au/heart-
health-education/blood-pressure-and-your-heart

Q.40
Walter Hendy is a 55 year old man who rarely attends your clinic. You note on his file that his past medical history lists an abdominal aortic aneurysm.
He tells you it was detected about a year ago when he had an ultrasound for another reason. You review the ultrasound report from 10 months ago and
see that the aneurysm measured 3cm at that time. He has no other past medical history on file and is on no regular medication. He is a smoker with a
30-pack year smoking history and his is blood pressure today is 160/105mmHg.

What are your primary management priorities for Walter and his abdominal aortic aneurysm? Choose one (1) option.

Recommend screening for all of Walter's first degree relatives for aortic aneurysms

Referral for an echocardiogram to exclude aortic root dilatation or ascending arch aneurysm

Optimise cardiovascular risk factors including hypertension and cholesterol and encourage smoking cessation

Refer Walter urgently to a vascular surgeon for advice on surgical management options

Put Walter on 6 monthly recalls for aneurysm surveillance to monitor size and progression
Your answer is correct.

General Feedback:
The correct response is: Optimise cardiovascular risk factors including hypertension and cholesterol and
encourage smoking cessation.
Medical management of an Abdominal Aortic aneurysm (AAA) generally involves cardiovascular risk
reduction, including antiplatelet therapy, statin therapy and antihypertensive therapy. This best medical
management, however, is generally not intended to limit expansion or reduce the size of the AAA. Managing
cardiovascular risk factors is crucial for improving the overall survival of patients and the outcomes of
future AAA repair. Smoking cessation should also be advised and encouraged in all patients with known
AAA on the basis that it increases life expectancy and reduces morbidity and mortality from aneurysm
repair. There is also evidence that smoking is associated with AAA development and increased aneurysm
expansion.
Traditionally, familial screening has been recommended for primary relatives over the age of 65 years, but
recent evidence suggests that this should also be extended to younger relatives in whom there is clinical
evidence of a collagen, elastin or connective tissue disorder.
It is suggested that only patients with a primary presentation of a large AAA should undergo
echocardiography to exclude aortic root dilatation and bicuspid aortic valve or ascending arch aneurysm.
Typically, surgical intervention has been recommended at thresholds of 5.5cm in men, and 5.0cm in women.
However, there seems to be a trend towards earlier intervention before this operative threshold in some
countries. Walter may need a vascular surgeon opinion in the future, however, this is not your most urgent
management priority at this stage.
In general, suggested surveillance intervals have been based on rupture risk and expected growth rates,
estimated from the size of the aneurysm. No surveillance guidelines have been published or endorsed by the
Australian and New Zealand Society for Vascular Surgery (ANZSVS) to date. The 2017 US Society for
Vascular Surgery (SVS) Guidelines recommend 10 yearly surveillance intervals for a 2.5-3.0cm aneurysm;
while the 2010 European Society for Vascular Surgery (ESVS) Guidelines recommend 24 month surveillance
intervals for a 3.0-3.9cm aneurysm.

References:
Chuen J and Theivendran M. Abdominal aortic aneurysm: an update. AJGP 2018; 47(5). https://www1.racgp.o
rg.au/ajgp/2018/may/aaa-an-update
Theivendran M. and Chuen J. Updates on AAA screening and surveillance. AJGP 2018; 47(5). https://www1.ra
cgp.org.au/ajgp/2018/may/aaa-screening-and-surveillance

Q 41
Q.41
Bill Cook is a 72-year-old male who presents with fatigue. He has a past medical history of an acute myocardial infarction at age 55. He states that over
the last 6 months he has become more tired with normal daily activities. He lives in a 2-storey house and gets breathless when climbing the stairs. He
denies any symptoms at rest and can dress himself without feeling short of breath. He states that he has put on 3 kg over the last 3 months and his
socks leave quite distinctive marks around his ankles. You arrange an echocardiogram, which demonstrates impaired left ventricular contractile
function (LVEF < 40%).
Which of the following medications improves outcomes in patients with heart failure with reduced ejection fraction (HFrEF)? Choose one (1) option.

Eplerenone

Verapamil

Frusemide

Bumetanide

Digoxin

Your answer is correct.

General Feedback:
The correct response is: Eplerenone
ACE inhibitors, angiotensin II receptor blockers, beta blockers, aldosterone antagonists and angiotensin-
receptor neprilysin inhibitors (ARNI) have been shown to improve outcomes in patients with heart failure
with reduced ejection fracture (HFrEF).
Reference:
eTG complete. Melbourne: Therapeutic Guidelines Limited 2019. Amended May 2022. Available from: https://t
gldcdp.tg.org.au/viewTopic?topicfile=heart-failure&guidelineName=Cardiovascular#toc_d1e402 (Requires
paid subscription) (Accessed November 2022).

Q 42
Q.42
Sarah is a 23 year old patient who presents complaining of two days of retrosternal pleuritic chest pain which is worse when she is lying in bed. She
recalls having a “cold” a few weeks ago but has otherwise been well recently. She denies any fever, cough, sputum, haemoptysis or limb swelling.
Sarah takes the oral contraceptive pill but takes no other medications. She has no significant past medical history.

On examination, she appears well perfused and non-distressed but anxious. She has a regular pulse but is mildly tachycardic. Her blood pressure is
120/80mmHg and oxygen saturations are 97% on room air. She has dual heart sounds and her chest is clear. Her calves are soft and non-tender. Her
ear, nose and throat examination is unremarkable.
You ask your practice nurse to perform an electrocardiogram which is below. What is the most likely diagnosis? Choose one (1) option.
Inferior ST elevation myocardial infarction

Acute pericarditis

Pulmonary embolism

Pneumothorax

Prolonged QT interval with tachycardia

Your answer is correct.

General Feedback:
The correct response is: Acute pericarditis.
The ECG shows the classic signs of acute pericarditis including sinus tachycardia, widespread ST elevation
and PR depression with reciprocal changes in aVR. Sarah’s pericarditis is likely due to the viral infection she
had a few weeks prior to the onset of her chest pain.
Reference:
Life in the Fast Lane. Pericarditis. Updated March 2021. https://litfl.com/pericarditis-ecg-library/ (Accessed
November 2022)
Image source:
Life in the Fast Lane. Pericarditis. Updated March 2019. Example 2. https://litfl.com/pericarditis-ecg-librar
y/ (Accessed November 2022)

Q.43
You are seeing Kath Jannes, a 62 year old female patient who consistently has high blood pressure readings in the clinic. She reassures you she just
has “white coat hypertension” and is not concerned about her blood pressure. Today her blood pressure is 162/97 mmHg.

You recommend that Kath performs home blood pressure readings using the clinic's automated machine which
is loaned out to patients. Kath is not keen to complete a 24 hour ambulatory blood pressure recording.
Which of the following statements about home blood pressure readings is correct? Choose one (1) option.

Patients should be advised to sit quietly for 1 minute before taking their blood pressure at home

Home blood pressure readings are less reliable than clinic readings in terms of determining true underlying blood
pressure

Patients should be advised to take one blood pressure measurement morning and evening for seven consecutive
days

Average home blood pressure readings of 135/85 mmHg or more is the threshold for diagnosing hypertension

Patients should be advised to take three blood pressure readings daily over a four week period

Your answer is correct.

General Feedback:
The correct response is: Average home blood pressure readings of 135/85 mmHg or more is the threshold
for diagnosing hypertension.
Home blood pressure (BP) monitoring is the self-measurement of BP in the home environment. It is
complementary to 24-hour ambulatory BP, for better diagnosis and management of patients with high BP.
Two measurements of home blood pressure should be taken, one minute apart, after 5 minutes of sitting
quietly, morning and evening, for 7 consecutive days.
Reference and Image source:
Sharman, J. et al. How to measure home blood pressure: recommendations for healthcare professionals and
patients. Australian Family Physician 2016; 45(1-2): 31-34.
https://www.racgp.org.au/afp/2016/januaryfebruary/how-to-measure-home-blood-pressure-recommendations
-for-healthcare-professionals-and-patients/

Q.44
Ryan is a 30 year old man who presents with chest pain for two days. He describes it as a constant sharp pain, worse on deep breathing and
coughing, with no radiation.

Ryan is normally fit and healthy. You review his past medical history and see he usually only comes in with coughs and colds. He was seen by a
colleague a week ago for a medical certificate when he had a sore throat.

Ryan says he is very active, and has been training for a triathlon, he wonders if he has pulled a chest muscle doing bench press at the gym.

On examination, he is afebrile with a respiratory rate of 20/min and SaO2 98% on room air. Heart sounds are dual. His chest is clear. His ECG shows
widespread ST elevation and PR segment depression.

What is the most likely diagnosis? Choose one (1) option.

Hyperkalaemia

Rhabdomyolysis

Pericarditis

ST elevation myocardial infarction

Musculoskeletal chest wall pain


Your answer is correct.

General Feedback:
The correct response is: "pericarditis"
The most common cause of pericarditis is idiopathic and viral. Patients often present with pleuritic chest
pain which is made worse by deep inspiration and lying supine. A pericardial friction rub may be detected on
physical examination but is only seen in about 35% of patients. The typical ECG findings include widespread
ST elevation with PR depression. This differs from an acute myocardial infarction where ST elevation is seen
in the coronary vascular territory and reciprocal PR depression is usually absent.

Reference:
Rahman A and Saraswat A. Pericarditis. Australian Family Physician 2017; 46 (11): 810-814.
https://www.racgp.org.au/afp/2017/november/pericarditis/

Q.45
Rowena Beer is a 52 year old who female presents for her annual health check. She is up-to-date with her mammograms and cervical screening tests.
There is no significant past medical history listed in her file. Her recent fasting blood tests were unremarkable but you note at her last visit that her
resting blood pressure was 155/92 mmHg. You repeat her blood pressure today on a manual sphygmomanometer and once again the reading is
slightly elevated at 158/91 mmHg. You double check that the cuff size is correct.

Which of the following is correct regarding Rowena’s blood pressure? Choose one (1) option.

The patient has Grade 1 (mild) hypertension

The patient has Grade 2 (moderate) hypertension

The patient has Grade 3 (severe) hypertension


The patient’s blood pressure is normal

The patient has high-normal blood pressure

Your answer is correct.

General Feedback:
Answer = The patient has Grade 1 (mild) hypertension

National Heart Foundation of Australia. Guideline for the diagnosis and management of hypertension in
adults – 2016. Melbourne: National Heart Foundation of Australia, 2016. Available from: https://www.heartfou
ndation.org.au/bundles/for-professionals/hypertension (Accessed Sept 2022)

Q.46
Nigel Thomas, a 71 year old man, has presented to your GP clinic with worsening shortness of breath when walking to his local supermarket and lying
flat in bed at night-time. Mr Thomas has a known history of type 2 diabetes mellitus and hypertension. His medications include metformin 1g orally
nocte and amlodipine 10mg orally mane.
His blood pressure is 172/96mmHg, with Mr Thomas reporting similar readings at home. You refer Mr Thomas for blood tests, an electrocardiogram, a
chest x-ray and echocardiogram to confirm your suspected diagnosis.
What medication change would make for Mr Thomas today? Choose one (1) option.

Commence digoxin 125mcg orally daily

Increase amlodipine to 10mg orally twice daily

Commence enalapril 2.5mg mane orally

Cease metformin 1g nocte, instead commencing empagliflozin 10mg orally nocte

Commence spironolactone 25mg mane orally

Your answer is correct.

General Feedback:
The correct answer is: “Commence enalapril 2.5mg mane orally”

This case is potentially an early presentation of heart failure that requires investigation with an
echocardiogram, prior to instituted treatment for heart failure. The key features are his breathlessness with
orthopnoea. However, Mr Thomas’ blood pressure needs addressing as well. Amlodipine is already at
maximum dosage and thus an additional agent is required. ACE-Inhibitors are preferred due to their benefit
with his potential heart failure, as such, a small dose should be commenced at this consultation.

Reference:
Atherton JJ, Sindone A, De Pasquale CG, Driscoll A, MacDonald PS, Hopper I, Kistler P, Briffa TG, Wong J,
Abhayaratna WP, Thomas L. National Heart Foundation of Australia and Cardiac Society of Australia and New
Zealand: Australian clinical guidelines for the management of heart failure 2018. Heart Lung & Circulation.
2018 Oct;27(10):1123-1208. https://www.heartlungcirc.org/article/S1443-9506(18)31777-3/fulltext (Accessed
September 2022)
eTG. Heart Failure. https://tgldcdp.tg.org.au/viewTopic?topicfile=heart-failure&guidelineName=Cardiovascula
r#toc_d1e238 (Subscription required). (Accessed September 2022).

Q.47
Warrah Ngale is a 42 year old indigenous woman who is your regular patient. Warrah works part time as a Lawyer in a private firm and is a single
mother to 3 teenage children. She has been seeing you recently for advice on preventing heart disease as she has a strong family history. Her father
had a myocardial infarction in his late 40s, her brother has type 2 diabetes and recently suffered a myocardial infarction aged 50. She is aware of more
distant relatives with similar issues.

Warrah reports feeling well and denies experiencing any symptoms of chest pain or breathlessness at rest or on exertion. She has tried
unsuccessfully to quit smoking over the years, and currently smokes 10-15 cigarettes per day. On examination today Warrah’s blood pressure today is
139/88mmHg and her BMI is 38kg/m2.

You arrange some investigations for Warrah including fasting bloods and coronary artery calcium. She returns to see you for her coronary artery
calcium score which returned a result of 150 AU. She has not had time to have her fasting bloods done yet.

Based on this result what treatment would you recommend for Warrah?

Please choose one (1) from the following options.

Recommend aspirin and clopidogrel for primary prevention of myocardial infarction

Recommend aspirin for primary prevention of myocardial infarction

Recommend statin therapy for primary prevention of myocardial infarction

Recommend both aspirin and statin therapy for primary prevention of myocardial infarction

Recommend metformin for primary prevention of myocardial infarction

Your answer is incorrect.


General Feedback:
The correct response is: Recommend both aspirin and statin therapy for primary prevention of myocardial
infarction
As per the article from Australian Journal of General Practice (AJGP) ‘Coronary artery calcium in primary
prevention’. Aspirin is not generally recommended for primary prevention as its cardiovascular benefits are
outweighed by an increased risk of bleeding. However, calcium scoring has been shown to identify a
subgroup of patients for whom prophylactic aspirin may be beneficial.
Using data from MESA, Miedema et al showed that aspirin had a net benefit among individuals with CAC
>100. Conversely, aspirin showed no net benefit for individuals with CAC = 0 and could not be universally
recommended for those with CAC = 1–99.
Studies have also assessed the ability of calcium scoring to guide lipid-lowering therapy. In a study of 950
individuals, the calculated five-year NNT of statins to prevent one coronary event was 549 for individuals
with CAC = 0, compared with 24 for those with CAC >100, suggesting that statins should be prescribed for
individuals with CAC >100 (irrespective of cholesterol levels) but may be withheld for those with CAC = 0.
These findings were recently corroborated in a large study of 13,644 individuals over 9.4 years of follow-up.
Reference:
Chua A, Blankstein R, Ko, B. Coronary artery calcium in primary prevention. AJGP 2020: 49(8). Available
from: https://www1.racgp.org.au/ajgp/2020/august/coronary-artery-calcium-in-primary-prevention (Accessed
August 2021).
Correct answers is: Recommend both aspirin and statin therapy for primary prevention of myocardial infarction

Q.48
Ken Garrahy is a 50 year old Caucasian man who is a regular patient of your practice. He has brought in an article from the local newspaper that he
recently read about coronary calcium scoring. He asks you whether this is a test that you would recommend for him. Ken tells you that he has never
smoked.

In which of the following scenarios would you consider ordering a Coronary Calcium Score for Ken? Choose one (1) option.

If Ken had a serum total cholesterol of 7.8 mmol/L.

If Ken’s father died of cardiovascular disease at age 40 and Ken had a total cholesterol of 4.2 mmol/L, high density
lipoprotein of 1.1 mmol/L , a blood pressure of 130/78mmHg and he did not have diabetes.

If Ken told you that he was having tightness in his chest when walking up a hill.

If Ken had type 2 diabetes with a urine albumin:creatinine ratio of 3.0 mg/mmol.

If Ken’s blood pressure was consistently around 165/90 mmHg despite being on perindopril 4mg orally daily.

Your answer is correct.

General Feedback:
The correct response is: If Ken’s father died of cardiovascular disease at age 40 and Ken had total
cholesterol of 4.2 mmol/L, high density lipoprotein of 1.1 mmol/L blood pressure of 130/78 mmHg and he did
not have diabetes.
Coronary calcium scoring is a type of low-radiation computed tomography (CT) scan which provides a
numerical value of a person’s coronary calcium level. This then correlates with the risk of having a
myocardial infarction or stroke in the next ten years. This test is of the highest value in asymptomatic
intermediate risk individuals or in individuals who are low risk according to the Framingham criteria but who
have additional risk factors that are not included in the calculation. For example, a family history of
premature coronary artery disease, a person with diabetes aged 40-60 years and Aboriginal and Torres Strait
Islander people age >40 years. The correct response describes an individual with a low risk of
cardiovascular disease according to the Framingham risk equation but with a first degree relative with
premature cardiovascular disease. Therefore, a coronary calcium score would be indicated in this case.
Three of the other options indicate patients already at high risk of cardiovascular disease.
For people with symptoms suggestive of myocardial ischaemia, an exercise stress test, CT coronary
angiogram or invasive angiogram is indicated.
References:
The Royal Australian College of General Practitioners. Guidelines for preventative activities in general
practice 9th edition [Internet]. Victoria. 2016. Pages 86-93. Available from: https://www.racgp.org.au/clinical-r
esources/clinical-guidelines/key-racgp-guidelines/view-all-racgp-guidelines/guidelines-for-preventive-activiti
es-in-general-pr/preamble/introduction (Accessed July 2021).
Chua A, Blankstein R, Ko R. Coronary artery calcium in primary prevention. Aust J Gen Pract [Internet]. 2020
Aus. Available from: https://www1.racgp.org.au/ajgp/2020/august/coronary-artery-calcium-in-primary-prevent
ion (Accessed July 2021).

Q.49
Rowena Porter is a 52 year old female who presents for her annual health check. Her recent fasting bloods were unremarkable but you note at her last
visit that her resting blood pressure was 155/92 mmHg. You repeat her blood pressure today and once again the reading is slightly elevated at 158/91
mmHg. She states that this is quite normal for her as she gets ‘White Coat Syndrome’.

Which statement is correct regarding measurement of Rowena’s blood pressure? Choose one (1) option.

Home blood pressure readings are recommended to be used in absolute cardiovascular risk score calculations.

Wrist home blood pressure monitors are recommended for patient use.
The criteria for diagnosing hypertension differs between clinic and ambulatory blood pressure readings.

Patients should avoid caffeine and smoking for at least 24 hours before blood pressure measurement.

Out-of-clinic blood pressure measurements are not essential to confirm a diagnosis of white-coat hypertension.

Your answer is correct.

General Feedback:
The correct response is: The criteria for diagnosing hypertension differs between clinic and ambulatory
blood pressure readings.
Detailed information regarding the correct measurement of blood pressure and interpretation of blood
pressure results can be found in the hypertension guidelines from the National Health Foundation.

Reference:
National Heart Foundation of Australia. Guideline for the diagnosis and management of hypertension in
adults - 2016. Melbourne: National Heart Foundation of Australia, 2016. Available from: https://www.heartfoun
dation.org.au/getmedia/c83511ab-835a-4fcf-96f5-88d770582ddc/PRO-167_Hypertension-guideline-2016_WEB.
pdf (Accessed September 2022).

Q.50
Sven, a 34 year old male presents with a history of chest pain, shortness of breath and fever. He states the pain is located over the left side of his
chest, is sharp in intensity and made worse by inspiration and lying flat. Your investigations confirm a diagnosis of pericarditis.

What is the most common overall cause of acute pericarditis in Australia? Choose one (1) option.
Medication-induced

Idiopathic

Autoimmune

Bacterial

Post-procedure

Your answer is correct.

General Feedback:
The correct response is: Idiopathic
Table 2 in the reference provides a list of the possible underlying causes of pericarditis, with idiopathic
being the most common. This is a diagnosis of exclusion. Viruses are the most common cause of infectious
pericardial disease.

Reference:
Rahman A, Sarasat A. Pericarditis. Aust Fam Physician 2017; 46(11): 810-814.
https://www.racgp.org.au/afp/2017/november/pericarditis/ (Accessed December 2022).

You might also like